Find stationary points of a two variable function involving

Click For Summary
The discussion focuses on finding stationary points of the function G(x, y) = (x^3)e^(-x^2-y^2). The partial derivatives Gx and Gy are calculated, leading to the equations 3x^2 - 2x^4 = 0 and (-2y)(x^3)e^(-x^2-y^2) = 0. The solutions yield stationary points at (0, a) for any real number a, and at (+/- √(3/2), 0). There is confusion regarding why Wolfram Alpha does not list the (0, a) points, while other software like Maple does recognize them. The conversation emphasizes the importance of verifying results across different computational tools.
Woolyabyss
Messages
142
Reaction score
1

Homework Statement


Find all stationary points of the function

G(x, y) = (x^3)*e^(−x^2−y^2)

Homework Equations


fx=0 and fy=0

The Attempt at a Solution



Gx = 3x^2*e^(-x^2-y^2) +x^3(-2x)e^(-x^2-y^2) = e^(-x^2-y^2)(3x^2-2x^4)

Gx = 0 implies 3x^2-2x^4=0

x^2(3-2x^2)=0

hence x =0 ,+or- (3/2)^(1/2)

Gy = (-2y)(x^3)(e^(-x^2-y^2))

Gy=0

implies (-2y)(x^3)(e^(-x^2-y^2))=0

y=0 x = 0

(+or- (3/2)^(1/2), 0) are the two stationary points according to wolfram alpha but I don't understand why (o,a), where a is a real number, isn't a solution since Gx=Gy=0 when x = 0.

Any help would be appreciated.
 
Last edited by a moderator:
Physics news on Phys.org
Woolyabyss said:

Homework Statement


Find all stationary points of the function

G(x, y) = (x^3)*e^(−x^2−y^2)

Homework Equations


fx=0 and fy=0

The Attempt at a Solution



Gx = 3x^2*e^(-x^2-y^2) +x^3(-2x)e^(-x^2-y^2) = e^(-x^2-y^2)(3x^2-2x^4)

Gx = 0 implies 3x^2-2x^4=0

x^2(3-2x^2)=0

hence x =0 ,+or- (3/2)^(1/2)

Gy = (-2y)(x^3)(e^(-x^2-y^2))

Gy=0

implies (-2y)(x^3)(e^(-x^2-y^2))=0

y=0 x = 0

(+or- (3/2)^(1/2), 0) are the two stationary points according to wolfram alpha but I don't understand why (o,a), where a is a real number, isn't a solution since Gx=Gy=0 when x = 0.

Any help would be appreciated.

The points (0,a) are solutions, but (according to you---I have not checked) Wolfram Alpha seems to have missed them. Maple did not miss them.
 
  • Like
Likes Woolyabyss
Question: A clock's minute hand has length 4 and its hour hand has length 3. What is the distance between the tips at the moment when it is increasing most rapidly?(Putnam Exam Question) Answer: Making assumption that both the hands moves at constant angular velocities, the answer is ## \sqrt{7} .## But don't you think this assumption is somewhat doubtful and wrong?

Similar threads

  • · Replies 2 ·
Replies
2
Views
2K
  • · Replies 18 ·
Replies
18
Views
3K
  • · Replies 10 ·
Replies
10
Views
2K
Replies
1
Views
1K
  • · Replies 6 ·
Replies
6
Views
2K
Replies
2
Views
1K
  • · Replies 5 ·
Replies
5
Views
2K
  • · Replies 10 ·
Replies
10
Views
2K
  • · Replies 8 ·
Replies
8
Views
2K
  • · Replies 10 ·
Replies
10
Views
2K